Difference between revisions of "1984 AIME Problems/Problem 8"

(Solution 2)
(23 intermediate revisions by 15 users not shown)
Line 1: Line 1:
 
== Problem ==
 
== Problem ==
The equation <math>\displaystyle z^6+z^3+1</math> has complex roots with argument <math>\theta</math> between <math>90^\circ</math> and <math>180^\circ</math> in thet complex plane. Determine the degree measure of <math>\theta</math>.
+
The equation <math>z^6+z^3+1=0</math> has [[complex root]]s with argument <math>\theta</math> between <math>90^\circ</math> and <math>180^\circ</math> in the [[complex plane]]. Determine the degree measure of <math>\theta</math>.
 +
 
 +
== Solution 1 ==
 +
We shall introduce another factor to make the equation easier to solve. If <math>r</math> is a root of <math>z^6+z^3+1</math>, then <math>0=(r^3-1)(r^6+r^3+1)=r^9-1</math>. The polynomial <math>x^9-1</math> has all of its roots with [[absolute value]] <math>1</math> and argument of the form <math>40m^\circ</math> for integer <math>m</math> (the ninth degree [[roots of unity]]). Now we simply need to find the root within the desired range that satisfies our original equation <math>x^6 + x^3 + 1 = 0</math>.
 +
 
 +
This reduces <math>\theta</math> to either <math>120^{\circ}</math> or <math>160^{\circ}</math>. But <math>\theta</math> can't be <math>120^{\circ}</math> because if <math>r=\cos 120^\circ +i\sin 120^\circ </math>, then <math>r^6+r^3+1=3</math>. (When we multiplied by <math>r^3 - 1</math> at the beginning, we introduced some extraneous solutions, and the solution with <math>120^\circ</math> was one of them.) This leaves <math>\boxed{\theta=160}</math>.
 +
 
 +
== Solution 2 ==
 +
The substitution <math>y=z^3</math> simplifies the equation to <math>y^2+y+1 = 0</math>. Applying the quadratic formula gives roots <math>y=-\frac{1}{2}\pm \frac{\sqrt{3}i}{2}</math>, which have arguments of <math>120</math> and <math>240,</math> respectively.
 +
We can write them as <math>z^3 = \cos 240^\circ + i\sin 240^\circ</math> and <math>z^3 = \cos 120^\circ + i\sin 120^\circ</math>.
 +
So we can use De Moivre's theorem (which I would suggest looking at if you never heard of it before) to find the fractional roots of the expressions above!
 +
For <math>\cos 240^\circ + i\sin 240</math> we have <math>(\cos 240^\circ + i\sin 240^\circ)^{1/3}</math> <math>\Rightarrow</math> <math>\cos 80^\circ + i\sin 80^\circ, \cos 200^\circ + i\sin200^\circ,</math> and <math>\cos 320^\circ + i\sin320^\circ.</math>
 +
Similarly for <math>(\cos 120^\circ + i\sin 120^\circ)^{1/3}</math>, we have <math>\cos 40^\circ + i\sin 40^\circ, \cos 160^\circ + i\sin 160^\circ,</math> and <math>\cos 280^\circ + i\sin 280^\circ.</math>
 +
The only argument out of all these roots that fits the description is <math>\theta = \boxed{160}</math>
 +
 
 +
~ blueballoon
  
== Solution ==
 
{{solution}}
 
 
== See also ==
 
== See also ==
* [[1984 AIME Problems/Problem 7 | Previous problem]]
+
{{AIME box|year=1984|num-b=7|num-a=9}}
* [[1984 AIME Problems/Problem 9 | Next problem]]
+
 
* [[1984 AIME Problems]]
+
[[Category:Intermediate Trigonometry Problems]]

Revision as of 14:19, 3 May 2021

Problem

The equation $z^6+z^3+1=0$ has complex roots with argument $\theta$ between $90^\circ$ and $180^\circ$ in the complex plane. Determine the degree measure of $\theta$.

Solution 1

We shall introduce another factor to make the equation easier to solve. If $r$ is a root of $z^6+z^3+1$, then $0=(r^3-1)(r^6+r^3+1)=r^9-1$. The polynomial $x^9-1$ has all of its roots with absolute value $1$ and argument of the form $40m^\circ$ for integer $m$ (the ninth degree roots of unity). Now we simply need to find the root within the desired range that satisfies our original equation $x^6 + x^3 + 1 = 0$.

This reduces $\theta$ to either $120^{\circ}$ or $160^{\circ}$. But $\theta$ can't be $120^{\circ}$ because if $r=\cos 120^\circ +i\sin 120^\circ$, then $r^6+r^3+1=3$. (When we multiplied by $r^3 - 1$ at the beginning, we introduced some extraneous solutions, and the solution with $120^\circ$ was one of them.) This leaves $\boxed{\theta=160}$.

Solution 2

The substitution $y=z^3$ simplifies the equation to $y^2+y+1 = 0$. Applying the quadratic formula gives roots $y=-\frac{1}{2}\pm \frac{\sqrt{3}i}{2}$, which have arguments of $120$ and $240,$ respectively. We can write them as $z^3 = \cos 240^\circ + i\sin 240^\circ$ and $z^3 = \cos 120^\circ + i\sin 120^\circ$. So we can use De Moivre's theorem (which I would suggest looking at if you never heard of it before) to find the fractional roots of the expressions above! For $\cos 240^\circ + i\sin 240$ we have $(\cos 240^\circ + i\sin 240^\circ)^{1/3}$ $\Rightarrow$ $\cos 80^\circ + i\sin 80^\circ, \cos 200^\circ + i\sin200^\circ,$ and $\cos 320^\circ + i\sin320^\circ.$ Similarly for $(\cos 120^\circ + i\sin 120^\circ)^{1/3}$, we have $\cos 40^\circ + i\sin 40^\circ, \cos 160^\circ + i\sin 160^\circ,$ and $\cos 280^\circ + i\sin 280^\circ.$ The only argument out of all these roots that fits the description is $\theta = \boxed{160}$

~ blueballoon

See also

1984 AIME (ProblemsAnswer KeyResources)
Preceded by
Problem 7
Followed by
Problem 9
1 2 3 4 5 6 7 8 9 10 11 12 13 14 15
All AIME Problems and Solutions